Question and Answers Forum

All Questions      Topic List

Differential Equation Questions

Previous in All Question      Next in All Question      

Previous in Differential Equation      Next in Differential Equation      

Question Number 105877 by john santu last updated on 01/Aug/20

Answered by bemath last updated on 01/Aug/20

(dy/dx).(1/x^3 ) + ((1+y^2 )/(y^4 (1+x^2 ))) = 0  ((y^4  dy)/(1+y^2 )) = −((x^3  dx)/(1+x^2 ))   ∫ ((y^4  dy)/(1+y^2 )) = −∫ ((x^3  dx)/(1+x^2 ))  let y = tan p and x = tan q   ∫ ((tan^4 p sec^2 p dp)/(sec^2 p)) = −∫((tan^3 q sec^2 q dq)/(sec^2 q))  ∫ tan^4 p dp = −∫ tan^3 q dq   ∫ tan^2 p (sec^2 p−1)dp = −∫tan q(sec^2 q−1)dq  I_1 = ∫ tan^2 p d(tan p)−∫d(tan p)+p  = (1/3)tan^3  p−tan p+p  = (1/3)y^3 −y+ tan^(−1) (y)  I_2 = −{∫tan q d(tan q)−∫ tan q dq}  I_2 =−(1/2)tan^2 q + ln ∣cos q∣   I_2 =−(1/2)x^2 + ln ∣(1/(√(1+x^2 )))∣   ∴ General solution  (1/3)y^3 −y+ tan^(−1) (y)=−(1/2)x^2 −ln(√(1+x^2 )) +C

$$\frac{{dy}}{{dx}}.\frac{\mathrm{1}}{{x}^{\mathrm{3}} }\:+\:\frac{\mathrm{1}+{y}^{\mathrm{2}} }{{y}^{\mathrm{4}} \left(\mathrm{1}+{x}^{\mathrm{2}} \right)}\:=\:\mathrm{0} \\ $$$$\frac{{y}^{\mathrm{4}} \:{dy}}{\mathrm{1}+{y}^{\mathrm{2}} }\:=\:−\frac{{x}^{\mathrm{3}} \:{dx}}{\mathrm{1}+{x}^{\mathrm{2}} }\: \\ $$$$\int\:\frac{{y}^{\mathrm{4}} \:{dy}}{\mathrm{1}+{y}^{\mathrm{2}} }\:=\:−\int\:\frac{{x}^{\mathrm{3}} \:{dx}}{\mathrm{1}+{x}^{\mathrm{2}} } \\ $$$${let}\:{y}\:=\:\mathrm{tan}\:{p}\:{and}\:{x}\:=\:\mathrm{tan}\:{q}\: \\ $$$$\int\:\frac{\mathrm{tan}\:^{\mathrm{4}} {p}\:\mathrm{sec}\:^{\mathrm{2}} {p}\:{dp}}{\mathrm{sec}\:^{\mathrm{2}} {p}}\:=\:−\int\frac{\mathrm{tan}\:^{\mathrm{3}} {q}\:\mathrm{sec}\:^{\mathrm{2}} {q}\:{dq}}{\mathrm{sec}\:^{\mathrm{2}} {q}} \\ $$$$\int\:\mathrm{tan}\:^{\mathrm{4}} {p}\:{dp}\:=\:−\int\:\mathrm{tan}\:^{\mathrm{3}} {q}\:{dq}\: \\ $$$$\int\:\mathrm{tan}\:^{\mathrm{2}} {p}\:\left(\mathrm{sec}\:^{\mathrm{2}} {p}−\mathrm{1}\right){dp}\:=\:−\int\mathrm{tan}\:{q}\left(\mathrm{sec}\:^{\mathrm{2}} {q}−\mathrm{1}\right){dq} \\ $$$${I}_{\mathrm{1}} =\:\int\:\mathrm{tan}\:^{\mathrm{2}} {p}\:{d}\left(\mathrm{tan}\:{p}\right)−\int{d}\left(\mathrm{tan}\:{p}\right)+{p} \\ $$$$=\:\frac{\mathrm{1}}{\mathrm{3}}\mathrm{tan}^{\mathrm{3}} \:{p}−\mathrm{tan}\:{p}+{p} \\ $$$$=\:\frac{\mathrm{1}}{\mathrm{3}}{y}^{\mathrm{3}} −{y}+\:\mathrm{tan}^{−\mathrm{1}} \left({y}\right) \\ $$$${I}_{\mathrm{2}} =\:−\left\{\int\mathrm{tan}\:{q}\:{d}\left(\mathrm{tan}\:{q}\right)−\int\:\mathrm{tan}\:{q}\:{dq}\right\} \\ $$$${I}_{\mathrm{2}} =−\frac{\mathrm{1}}{\mathrm{2}}\mathrm{tan}\:^{\mathrm{2}} {q}\:+\:\mathrm{ln}\:\mid\mathrm{cos}\:{q}\mid\: \\ $$$${I}_{\mathrm{2}} =−\frac{\mathrm{1}}{\mathrm{2}}{x}^{\mathrm{2}} +\:\mathrm{ln}\:\mid\frac{\mathrm{1}}{\sqrt{\mathrm{1}+{x}^{\mathrm{2}} }}\mid\: \\ $$$$\therefore\:\mathcal{G}{eneral}\:{solution} \\ $$$$\frac{\mathrm{1}}{\mathrm{3}}{y}^{\mathrm{3}} −{y}+\:\mathrm{tan}^{−\mathrm{1}} \left({y}\right)=−\frac{\mathrm{1}}{\mathrm{2}}{x}^{\mathrm{2}} −\mathrm{ln}\sqrt{\mathrm{1}+{x}^{\mathrm{2}} }\:+{C}\: \\ $$

Terms of Service

Privacy Policy

Contact: info@tinkutara.com